¿Una transformación de calibre deja la física invariante?

Se suele decir que la invariancia de calibre es capturada por la invariancia de los campos eléctricos y magnéticos en la siguiente transformación de los potenciales:

ϕ = ϕ x t A = A + x

En mecánica cuántica, considerando un hamiltoniano en la forma

H = 1 2 metro ( PAG q A ) 2 + q ϕ ,

el efecto de la transformación de calibre se puede cancelar transformando la función de onda en

ψ = mi i q x ψ .

¿Tengo razón hasta ahora?

Si lo soy porque se supone que la transformación de calibre no cambia nada, significa que cada expectativa se puede calcular de manera equivalente usando ψ o ψ , lo cual no parece no ser cierto.

Un ejemplo podría ser una partícula en un mi = B = 0 región. Entonces puedo elegir ϕ = A = 0 , para hacer el hamiltoniano

H = 1 2 metro PAG 2

Pero uno de mis amigos podría haber seleccionado ϕ = 0 , A = a , que se supone que es equivalente a mi elección ya que están vinculados por una transformación de calibre ( x = a X ). Entonces, el hamiltoniano sería

H = 1 2 metro ( PAG q a ) 2

La función de onda de calibre transformada que puede tomar para hacer que su hamiltoniano sea similar al mío es ψ = mi i q a X ψ , pero esta función no es equivalente.

El valor esperado del impulso para mí es:

PAG ψ = i ψ X ψ d X

Para mi amigo es:

PAG ψ = i mi i q a X ψ X ( mi i q a X ψ ) d X = q a ψ ψ d X i ψ X ψ d X

¿Qué es lo que no entiendo?

¿Qué crees que significa "equivalente" en este contexto?
Trate de dar un ejemplo de valores esperados que no coincidan.
La multiplicación de una función de onda por una fase general como lo haría en una 'transformación de calibre' no hace ninguna diferencia en los cálculos, ya que se cancela por su complejo conjugado cada vez que calcula un valor esperado.
También tenga en cuenta que el impulso que aparece en el hamiltoniano es un impulso canónico. L / X ˙ , no momento mecánico metro v .
Agregué el ejemplo del valor esperado del impulso, pero me hace pensar que tal vez el impulso correcto que debería usar para el valor esperado es p - qa. Si este es mi error, ¿alguien puede responder dando la regla general de cómo deben transformarse los observables?
@Javier sí, pero no del todo: el PAG en el hamiltoniano es el momento canónico, pero el hamiltoniano sigue siendo ( metro v ) 2 / 2 metro , es decir, el momento cinético expresado en función del momento canónico.
@Emilio: No dije eso, aunque tal vez debería haber aclarado el punto.

Respuestas (2)

La declaración

debido a que se supone que la transformación de calibre no cambia nada, significa que cada expectativa se puede calcular de manera equivalente usando ψ o ψ

no es particularmente correcto. Todos los valores esperados medibles físicamente pueden calcularse correctamente en cualquier indicador arbitrario, pero la representación de algunos operadores puede cambiar de un indicador a otro.

El punto de fricción particular aquí es la diferencia entre el momento cinético y el momento canónico . Para ser claro,

  • El momento cinético es siempre igual a metro v , es físicamente medible y, por lo tanto, debe ser invariante de calibre, mientras que
  • El momento canónico es la cantidad pag tal que [ X ^ i , pag ^ j ] = i d i j (por lo tanto, su representación de posición es siempre el gradiente, es decir X | pag ^ j = i X j X | ), y no es invariante de calibre. Como ejemplo, en presencia de un campo electromagnético, el momento canónico generalmente difiere del momento cinético por un múltiplo del vector potencial A .

Cuando cambia los indicadores, es importante tener cuidado de transformar todos los observables relevantes en el proceso; el hecho de no hacerlo es probablemente la razón más común de confusión con respecto a las dependencias de calibre espurias en los resultados.

También es importante tener en cuenta que nada de esto es específico de la mecánica cuántica, y obtienes exactamente los mismos problemas y conceptos desde los comienzos de la mecánica lagrangiana, desde calibres y cambios de calibre hasta la falta de coincidencia entre el momento canónico y cinético. Para obtener más información, consulte su libro de texto de mecánica analítica favorito.

Tiene sentido. ¿Hay alguna manera de hacer que toda esta transformación de calibre sea más "mecánica cuántica" (con eso quiero decir no tener que referirme a la mecánica lagrangiana) mediante la introducción de algún tipo de operador unitario asociado con una transformación de calibre?
Sí, y ya lo has hecho, es el factor de tu ^ = mi i q x ( r ^ ) . Sin embargo, tratar de desacoplar QM por completo y olvidar su base y sus análogos en la mecánica lagrangiana es una idea completamente mala (y las declaraciones engañosas de la firma "X es esta nueva característica inestable que es exclusivamente mecánica cuántica" son particularmente perniciosas cuando son falsas, como lo es el caso de las transformaciones de calibre). Lo que está pidiendo es simplemente la forma formalmente correcta de hacer esto en QM.
Gracias, tienes razón. Supongo que el siguiente paso para mí es obtener una comprensión más general de cómo funcionan las simetrías en QM.

Tienes razón. El impulso operativo no es invariante de calibre. Si exige que las cantidades medibles sean invariantes de calibre (es un requisito sensato), el impulso no es una cantidad medible. La velocidad es sin embargo. El único libro que pude encontrar mencionando esto es Ballentines. Ballentine 'Quantum Mechanics' afirma en el Capítulo 11: "Dado que p, como A, se cambia por una transformación de calibre, también carece de un significado físico directo". (luego siguen algunos cálculos sobre el operador de velocidad)